Solve the given inequality below in the body.Rational/Quadratic InequalityProof that $left(sum^n_k=1x_kright)left(sum^n_k=1y_kright)geq n^2$Is $int_x^inftye^-fract^22 < frac1xe^-fracx^22$?Is the inequality solution below legal?Young's inequality or something similarTypical Absolute value inequalityWhich is the proper way to solve the inequality problems?Solve the inequality $sin x > ln x$

What's the point of this macro?

Fantasy Military Arms and Armor: the Dwarven Grand Armory

Is there any reason to change the ISO manually?

What's the difference between a share and a stock?

Is directly echoing the user agent in PHP a security hole?

How to find better food in airports

Why there is no wireless switch?

How were the names on the memorial stones in Avengers: Endgame chosen, out-of-universe?

If magnetic force can't do any work, then how can we define a potential?

In-universe, why does Doc Brown program the time machine to go to 1955?

First Number to Contain Each Letter

Is the interior of a Bag of Holding actually an extradimensional space?

Why does the seven segment display have decimal point at the right?

What is the source of the fear in the Hallow spell's extra Fear effect?

Tying double knot of garbarge bag

Re-running Calculate Field tool in ArcGIS Pro?

Is it possible to observe space debris with Binoculars?

Comparing elements in a nested list to generate a new list

A Meal fit for a King

'Hard work never hurt anyone' Why not 'hurts'?

Never make public members virtual/abstract - really?

What are some countries where you can be imprisoned for reading or owning a Bible?

Why don't they build airplanes from 3D printer plastic?

How many people can lift Thor's hammer?



Solve the given inequality below in the body.


Rational/Quadratic InequalityProof that $left(sum^n_k=1x_kright)left(sum^n_k=1y_kright)geq n^2$Is $int_x^inftye^-fract^22 < frac1xe^-fracx^22$?Is the inequality solution below legal?Young's inequality or something similarTypical Absolute value inequalityWhich is the proper way to solve the inequality problems?Solve the inequality $sin x > ln x$






.everyoneloves__top-leaderboard:empty,.everyoneloves__mid-leaderboard:empty,.everyoneloves__bot-mid-leaderboard:empty margin-bottom:0;








4












$begingroup$



$$frac1-3 le frac 12$$




Let’s consider $|x|=y$



So $$frac1y-3-frac 12 le 0$$



$$frac2-y+3y-3 le 0$$
$$fracy-5y-3 ge 0$$
$$y in (-infty , 3)cup [5, infty)$$
Now this is where the problem starts. I cannot figure out on how to break the modulus function, so that’s the part where I need help in. Thanks a lot!










share|cite|improve this question









$endgroup$




















    4












    $begingroup$



    $$frac1-3 le frac 12$$




    Let’s consider $|x|=y$



    So $$frac1y-3-frac 12 le 0$$



    $$frac2-y+3y-3 le 0$$
    $$fracy-5y-3 ge 0$$
    $$y in (-infty , 3)cup [5, infty)$$
    Now this is where the problem starts. I cannot figure out on how to break the modulus function, so that’s the part where I need help in. Thanks a lot!










    share|cite|improve this question









    $endgroup$
















      4












      4








      4


      1



      $begingroup$



      $$frac1-3 le frac 12$$




      Let’s consider $|x|=y$



      So $$frac1y-3-frac 12 le 0$$



      $$frac2-y+3y-3 le 0$$
      $$fracy-5y-3 ge 0$$
      $$y in (-infty , 3)cup [5, infty)$$
      Now this is where the problem starts. I cannot figure out on how to break the modulus function, so that’s the part where I need help in. Thanks a lot!










      share|cite|improve this question









      $endgroup$





      $$frac1-3 le frac 12$$




      Let’s consider $|x|=y$



      So $$frac1y-3-frac 12 le 0$$



      $$frac2-y+3y-3 le 0$$
      $$fracy-5y-3 ge 0$$
      $$y in (-infty , 3)cup [5, infty)$$
      Now this is where the problem starts. I cannot figure out on how to break the modulus function, so that’s the part where I need help in. Thanks a lot!







      inequality






      share|cite|improve this question













      share|cite|improve this question











      share|cite|improve this question




      share|cite|improve this question










      asked 8 hours ago









      Aditya Aditya

      39410 bronze badges




      39410 bronze badges























          3 Answers
          3






          active

          oldest

          votes


















          4














          $begingroup$

          Anyway, in your case, $yge 0$, so you should transform all this to
          $$|x|<3;textit or ; |x|ge 5iff -3<x<3;textit or ; xge 5;textit or ; xle -5$$
          or, as a set, the union of three intervals:
          $$(-infty,-5]cup(-3,3)cup[5,+infty). $$






          share|cite|improve this answer









          $endgroup$






















            2














            $begingroup$

            The equivalences $$lvert Arvert< BiffbegincasesBge 0\ A> -B\ A< Bendcases\ lvert Arvertge Biff Ble 0lor begincasesB> 0\ Ale -BendcaseslorbegincasesB>0\ Age Bendcases$$



            are a good place to start from.






            share|cite|improve this answer











            $endgroup$














            • $begingroup$
              In your first equivalence, the inequalities should be strict.
              $endgroup$
              – Bernard
              8 hours ago










            • $begingroup$
              @Bernard Yes, thank you.
              $endgroup$
              – Gae. S.
              8 hours ago










            • $begingroup$
              Even $B$ should be positive, since $|A|ge 0$.
              $endgroup$
              – Bernard
              8 hours ago










            • $begingroup$
              @Bernard For $B=0$ the other two inequations sort it out (though you may argue that so do they when $B<0$).
              $endgroup$
              – Gae. S.
              8 hours ago










            • $begingroup$
              There remains that $|A|<BRightarrow B>0$, from a strictly logical point of view.
              $endgroup$
              – Bernard
              8 hours ago


















            1














            $begingroup$

            For $|x|<3$, the LHS is negative and the inequation certainly holds. Now assuming $|x|>3$, we can solve the equation



            $$frac1-3=frac12,$$ which gives $|x|=5$. The inequation holds for $|x|ge5$. Your resolution was right.



            So the solution set is



            $$(-3,3)cup(-infty,-5]cup[5,infty).$$






            share|cite|improve this answer











            $endgroup$

















              Your Answer








              StackExchange.ready(function()
              var channelOptions =
              tags: "".split(" "),
              id: "69"
              ;
              initTagRenderer("".split(" "), "".split(" "), channelOptions);

              StackExchange.using("externalEditor", function()
              // Have to fire editor after snippets, if snippets enabled
              if (StackExchange.settings.snippets.snippetsEnabled)
              StackExchange.using("snippets", function()
              createEditor();
              );

              else
              createEditor();

              );

              function createEditor()
              StackExchange.prepareEditor(
              heartbeatType: 'answer',
              autoActivateHeartbeat: false,
              convertImagesToLinks: true,
              noModals: true,
              showLowRepImageUploadWarning: true,
              reputationToPostImages: 10,
              bindNavPrevention: true,
              postfix: "",
              imageUploader:
              brandingHtml: "Powered by u003ca class="icon-imgur-white" href="https://imgur.com/"u003eu003c/au003e",
              contentPolicyHtml: "User contributions licensed under u003ca href="https://creativecommons.org/licenses/by-sa/3.0/"u003ecc by-sa 3.0 with attribution requiredu003c/au003e u003ca href="https://stackoverflow.com/legal/content-policy"u003e(content policy)u003c/au003e",
              allowUrls: true
              ,
              noCode: true, onDemand: true,
              discardSelector: ".discard-answer"
              ,immediatelyShowMarkdownHelp:true
              );



              );













              draft saved

              draft discarded


















              StackExchange.ready(
              function ()
              StackExchange.openid.initPostLogin('.new-post-login', 'https%3a%2f%2fmath.stackexchange.com%2fquestions%2f3344323%2fsolve-the-given-inequality-below-in-the-body%23new-answer', 'question_page');

              );

              Post as a guest















              Required, but never shown

























              3 Answers
              3






              active

              oldest

              votes








              3 Answers
              3






              active

              oldest

              votes









              active

              oldest

              votes






              active

              oldest

              votes









              4














              $begingroup$

              Anyway, in your case, $yge 0$, so you should transform all this to
              $$|x|<3;textit or ; |x|ge 5iff -3<x<3;textit or ; xge 5;textit or ; xle -5$$
              or, as a set, the union of three intervals:
              $$(-infty,-5]cup(-3,3)cup[5,+infty). $$






              share|cite|improve this answer









              $endgroup$



















                4














                $begingroup$

                Anyway, in your case, $yge 0$, so you should transform all this to
                $$|x|<3;textit or ; |x|ge 5iff -3<x<3;textit or ; xge 5;textit or ; xle -5$$
                or, as a set, the union of three intervals:
                $$(-infty,-5]cup(-3,3)cup[5,+infty). $$






                share|cite|improve this answer









                $endgroup$

















                  4














                  4










                  4







                  $begingroup$

                  Anyway, in your case, $yge 0$, so you should transform all this to
                  $$|x|<3;textit or ; |x|ge 5iff -3<x<3;textit or ; xge 5;textit or ; xle -5$$
                  or, as a set, the union of three intervals:
                  $$(-infty,-5]cup(-3,3)cup[5,+infty). $$






                  share|cite|improve this answer









                  $endgroup$



                  Anyway, in your case, $yge 0$, so you should transform all this to
                  $$|x|<3;textit or ; |x|ge 5iff -3<x<3;textit or ; xge 5;textit or ; xle -5$$
                  or, as a set, the union of three intervals:
                  $$(-infty,-5]cup(-3,3)cup[5,+infty). $$







                  share|cite|improve this answer












                  share|cite|improve this answer



                  share|cite|improve this answer










                  answered 8 hours ago









                  BernardBernard

                  132k7 gold badges43 silver badges126 bronze badges




                  132k7 gold badges43 silver badges126 bronze badges


























                      2














                      $begingroup$

                      The equivalences $$lvert Arvert< BiffbegincasesBge 0\ A> -B\ A< Bendcases\ lvert Arvertge Biff Ble 0lor begincasesB> 0\ Ale -BendcaseslorbegincasesB>0\ Age Bendcases$$



                      are a good place to start from.






                      share|cite|improve this answer











                      $endgroup$














                      • $begingroup$
                        In your first equivalence, the inequalities should be strict.
                        $endgroup$
                        – Bernard
                        8 hours ago










                      • $begingroup$
                        @Bernard Yes, thank you.
                        $endgroup$
                        – Gae. S.
                        8 hours ago










                      • $begingroup$
                        Even $B$ should be positive, since $|A|ge 0$.
                        $endgroup$
                        – Bernard
                        8 hours ago










                      • $begingroup$
                        @Bernard For $B=0$ the other two inequations sort it out (though you may argue that so do they when $B<0$).
                        $endgroup$
                        – Gae. S.
                        8 hours ago










                      • $begingroup$
                        There remains that $|A|<BRightarrow B>0$, from a strictly logical point of view.
                        $endgroup$
                        – Bernard
                        8 hours ago















                      2














                      $begingroup$

                      The equivalences $$lvert Arvert< BiffbegincasesBge 0\ A> -B\ A< Bendcases\ lvert Arvertge Biff Ble 0lor begincasesB> 0\ Ale -BendcaseslorbegincasesB>0\ Age Bendcases$$



                      are a good place to start from.






                      share|cite|improve this answer











                      $endgroup$














                      • $begingroup$
                        In your first equivalence, the inequalities should be strict.
                        $endgroup$
                        – Bernard
                        8 hours ago










                      • $begingroup$
                        @Bernard Yes, thank you.
                        $endgroup$
                        – Gae. S.
                        8 hours ago










                      • $begingroup$
                        Even $B$ should be positive, since $|A|ge 0$.
                        $endgroup$
                        – Bernard
                        8 hours ago










                      • $begingroup$
                        @Bernard For $B=0$ the other two inequations sort it out (though you may argue that so do they when $B<0$).
                        $endgroup$
                        – Gae. S.
                        8 hours ago










                      • $begingroup$
                        There remains that $|A|<BRightarrow B>0$, from a strictly logical point of view.
                        $endgroup$
                        – Bernard
                        8 hours ago













                      2














                      2










                      2







                      $begingroup$

                      The equivalences $$lvert Arvert< BiffbegincasesBge 0\ A> -B\ A< Bendcases\ lvert Arvertge Biff Ble 0lor begincasesB> 0\ Ale -BendcaseslorbegincasesB>0\ Age Bendcases$$



                      are a good place to start from.






                      share|cite|improve this answer











                      $endgroup$



                      The equivalences $$lvert Arvert< BiffbegincasesBge 0\ A> -B\ A< Bendcases\ lvert Arvertge Biff Ble 0lor begincasesB> 0\ Ale -BendcaseslorbegincasesB>0\ Age Bendcases$$



                      are a good place to start from.







                      share|cite|improve this answer














                      share|cite|improve this answer



                      share|cite|improve this answer








                      edited 8 hours ago

























                      answered 8 hours ago









                      Gae. S.Gae. S.

                      2,0648 silver badges17 bronze badges




                      2,0648 silver badges17 bronze badges














                      • $begingroup$
                        In your first equivalence, the inequalities should be strict.
                        $endgroup$
                        – Bernard
                        8 hours ago










                      • $begingroup$
                        @Bernard Yes, thank you.
                        $endgroup$
                        – Gae. S.
                        8 hours ago










                      • $begingroup$
                        Even $B$ should be positive, since $|A|ge 0$.
                        $endgroup$
                        – Bernard
                        8 hours ago










                      • $begingroup$
                        @Bernard For $B=0$ the other two inequations sort it out (though you may argue that so do they when $B<0$).
                        $endgroup$
                        – Gae. S.
                        8 hours ago










                      • $begingroup$
                        There remains that $|A|<BRightarrow B>0$, from a strictly logical point of view.
                        $endgroup$
                        – Bernard
                        8 hours ago
















                      • $begingroup$
                        In your first equivalence, the inequalities should be strict.
                        $endgroup$
                        – Bernard
                        8 hours ago










                      • $begingroup$
                        @Bernard Yes, thank you.
                        $endgroup$
                        – Gae. S.
                        8 hours ago










                      • $begingroup$
                        Even $B$ should be positive, since $|A|ge 0$.
                        $endgroup$
                        – Bernard
                        8 hours ago










                      • $begingroup$
                        @Bernard For $B=0$ the other two inequations sort it out (though you may argue that so do they when $B<0$).
                        $endgroup$
                        – Gae. S.
                        8 hours ago










                      • $begingroup$
                        There remains that $|A|<BRightarrow B>0$, from a strictly logical point of view.
                        $endgroup$
                        – Bernard
                        8 hours ago















                      $begingroup$
                      In your first equivalence, the inequalities should be strict.
                      $endgroup$
                      – Bernard
                      8 hours ago




                      $begingroup$
                      In your first equivalence, the inequalities should be strict.
                      $endgroup$
                      – Bernard
                      8 hours ago












                      $begingroup$
                      @Bernard Yes, thank you.
                      $endgroup$
                      – Gae. S.
                      8 hours ago




                      $begingroup$
                      @Bernard Yes, thank you.
                      $endgroup$
                      – Gae. S.
                      8 hours ago












                      $begingroup$
                      Even $B$ should be positive, since $|A|ge 0$.
                      $endgroup$
                      – Bernard
                      8 hours ago




                      $begingroup$
                      Even $B$ should be positive, since $|A|ge 0$.
                      $endgroup$
                      – Bernard
                      8 hours ago












                      $begingroup$
                      @Bernard For $B=0$ the other two inequations sort it out (though you may argue that so do they when $B<0$).
                      $endgroup$
                      – Gae. S.
                      8 hours ago




                      $begingroup$
                      @Bernard For $B=0$ the other two inequations sort it out (though you may argue that so do they when $B<0$).
                      $endgroup$
                      – Gae. S.
                      8 hours ago












                      $begingroup$
                      There remains that $|A|<BRightarrow B>0$, from a strictly logical point of view.
                      $endgroup$
                      – Bernard
                      8 hours ago




                      $begingroup$
                      There remains that $|A|<BRightarrow B>0$, from a strictly logical point of view.
                      $endgroup$
                      – Bernard
                      8 hours ago











                      1














                      $begingroup$

                      For $|x|<3$, the LHS is negative and the inequation certainly holds. Now assuming $|x|>3$, we can solve the equation



                      $$frac1-3=frac12,$$ which gives $|x|=5$. The inequation holds for $|x|ge5$. Your resolution was right.



                      So the solution set is



                      $$(-3,3)cup(-infty,-5]cup[5,infty).$$






                      share|cite|improve this answer











                      $endgroup$



















                        1














                        $begingroup$

                        For $|x|<3$, the LHS is negative and the inequation certainly holds. Now assuming $|x|>3$, we can solve the equation



                        $$frac1-3=frac12,$$ which gives $|x|=5$. The inequation holds for $|x|ge5$. Your resolution was right.



                        So the solution set is



                        $$(-3,3)cup(-infty,-5]cup[5,infty).$$






                        share|cite|improve this answer











                        $endgroup$

















                          1














                          1










                          1







                          $begingroup$

                          For $|x|<3$, the LHS is negative and the inequation certainly holds. Now assuming $|x|>3$, we can solve the equation



                          $$frac1-3=frac12,$$ which gives $|x|=5$. The inequation holds for $|x|ge5$. Your resolution was right.



                          So the solution set is



                          $$(-3,3)cup(-infty,-5]cup[5,infty).$$






                          share|cite|improve this answer











                          $endgroup$



                          For $|x|<3$, the LHS is negative and the inequation certainly holds. Now assuming $|x|>3$, we can solve the equation



                          $$frac1-3=frac12,$$ which gives $|x|=5$. The inequation holds for $|x|ge5$. Your resolution was right.



                          So the solution set is



                          $$(-3,3)cup(-infty,-5]cup[5,infty).$$







                          share|cite|improve this answer














                          share|cite|improve this answer



                          share|cite|improve this answer








                          edited 6 hours ago

























                          answered 8 hours ago









                          Yves DaoustYves Daoust

                          145k10 gold badges89 silver badges248 bronze badges




                          145k10 gold badges89 silver badges248 bronze badges






























                              draft saved

                              draft discarded
















































                              Thanks for contributing an answer to Mathematics Stack Exchange!


                              • Please be sure to answer the question. Provide details and share your research!

                              But avoid


                              • Asking for help, clarification, or responding to other answers.

                              • Making statements based on opinion; back them up with references or personal experience.

                              Use MathJax to format equations. MathJax reference.


                              To learn more, see our tips on writing great answers.




                              draft saved


                              draft discarded














                              StackExchange.ready(
                              function ()
                              StackExchange.openid.initPostLogin('.new-post-login', 'https%3a%2f%2fmath.stackexchange.com%2fquestions%2f3344323%2fsolve-the-given-inequality-below-in-the-body%23new-answer', 'question_page');

                              );

                              Post as a guest















                              Required, but never shown





















































                              Required, but never shown














                              Required, but never shown












                              Required, but never shown







                              Required, but never shown

































                              Required, but never shown














                              Required, but never shown












                              Required, but never shown







                              Required, but never shown







                              Popular posts from this blog

                              Invision Community Contents History See also References External links Navigation menuProprietaryinvisioncommunity.comIPS Community ForumsIPS Community Forumsthis blog entry"License Changes, IP.Board 3.4, and the Future""Interview -- Matt Mecham of Ibforums""CEO Invision Power Board, Matt Mecham Is a Liar, Thief!"IPB License Explanation 1.3, 1.3.1, 2.0, and 2.1ArchivedSecurity Fixes, Updates And Enhancements For IPB 1.3.1Archived"New Demo Accounts - Invision Power Services"the original"New Default Skin"the original"Invision Power Board 3.0.0 and Applications Released"the original"Archived copy"the original"Perpetual licenses being done away with""Release Notes - Invision Power Services""Introducing: IPS Community Suite 4!"Invision Community Release Notes

                              Canceling a color specificationRandomly assigning color to Graphics3D objects?Default color for Filling in Mathematica 9Coloring specific elements of sets with a prime modified order in an array plotHow to pick a color differing significantly from the colors already in a given color list?Detection of the text colorColor numbers based on their valueCan color schemes for use with ColorData include opacity specification?My dynamic color schemes

                              Tom Holland Mục lục Đầu đời và giáo dục | Sự nghiệp | Cuộc sống cá nhân | Phim tham gia | Giải thưởng và đề cử | Chú thích | Liên kết ngoài | Trình đơn chuyển hướngProfile“Person Details for Thomas Stanley Holland, "England and Wales Birth Registration Index, 1837-2008" — FamilySearch.org”"Meet Tom Holland... the 16-year-old star of The Impossible""Schoolboy actor Tom Holland finds himself in Oscar contention for role in tsunami drama"“Naomi Watts on the Prince William and Harry's reaction to her film about the late Princess Diana”lưu trữ"Holland and Pflueger Are West End's Two New 'Billy Elliots'""I'm so envious of my son, the movie star! British writer Dominic Holland's spent 20 years trying to crack Hollywood - but he's been beaten to it by a very unlikely rival"“Richard and Margaret Povey of Jersey, Channel Islands, UK: Information about Thomas Stanley Holland”"Tom Holland to play Billy Elliot""New Billy Elliot leaving the garage"Billy Elliot the Musical - Tom Holland - Billy"A Tale of four Billys: Tom Holland""The Feel Good Factor""Thames Christian College schoolboys join Myleene Klass for The Feelgood Factor""Government launches £600,000 arts bursaries pilot""BILLY's Chapman, Holland, Gardner & Jackson-Keen Visit Prime Minister""Elton John 'blown away' by Billy Elliot fifth birthday" (video with John's interview and fragments of Holland's performance)"First News interviews Arrietty's Tom Holland"“33rd Critics' Circle Film Awards winners”“National Board of Review Current Awards”Bản gốc"Ron Howard Whaling Tale 'In The Heart Of The Sea' Casts Tom Holland"“'Spider-Man' Finds Tom Holland to Star as New Web-Slinger”lưu trữ“Captain America: Civil War (2016)”“Film Review: ‘Captain America: Civil War’”lưu trữ“‘Captain America: Civil War’ review: Choose your own avenger”lưu trữ“The Lost City of Z reviews”“Sony Pictures and Marvel Studios Find Their 'Spider-Man' Star and Director”“‘Mary Magdalene’, ‘Current War’ & ‘Wind River’ Get 2017 Release Dates From Weinstein”“Lionsgate Unleashing Daisy Ridley & Tom Holland Starrer ‘Chaos Walking’ In Cannes”“PTA's 'Master' Leads Chicago Film Critics Nominations, UPDATED: Houston and Indiana Critics Nominations”“Nominaciones Goya 2013 Telecinco Cinema – ENG”“Jameson Empire Film Awards: Martin Freeman wins best actor for performance in The Hobbit”“34th Annual Young Artist Awards”Bản gốc“Teen Choice Awards 2016—Captain America: Civil War Leads Second Wave of Nominations”“BAFTA Film Award Nominations: ‘La La Land’ Leads Race”“Saturn Awards Nominations 2017: 'Rogue One,' 'Walking Dead' Lead”Tom HollandTom HollandTom HollandTom Hollandmedia.gettyimages.comWorldCat Identities300279794no20130442900000 0004 0355 42791085670554170004732cb16706349t(data)XX5557367